اہم مواد پر چھوڑ دیں
Microsoft
|
Math Solver
حل کرنا
عادت
کھیلنا
موضوعات
قبل از الجبرا
چھوٹا
روپ
سب سے بڑا عام عنصر
کم سے کم عام ملٹی پل
آرڈر آف آپریشنز
جزوی طور پر
مخلوط اجزاء
پرائم فیکٹرائزیشن
Exponents
بنیاد پرست
الجبرا
جیسی شرائط کو یکجا کریں
متغیر کے لئے حل کریں
عنصر
توسیع
اجزاء کا جائزہ لیں
لکیری مساوات
دوطرفہ مساوات
عدم مساوات
مساوات کے نظام
میٹرکس
Trigonometry
آسان بنائیں
اندازہ
Graphs
مساوات کو حل کریں
حسابان
ڈیریویٹوز
لازمی طور پر
حدود
الجبرا ان پٹ
Trigonometry Inputs
کیلکولس ان پٹ
Matrix ان پٹ
حل کرنا
عادت
کھیلنا
موضوعات
قبل از الجبرا
چھوٹا
روپ
سب سے بڑا عام عنصر
کم سے کم عام ملٹی پل
آرڈر آف آپریشنز
جزوی طور پر
مخلوط اجزاء
پرائم فیکٹرائزیشن
Exponents
بنیاد پرست
الجبرا
جیسی شرائط کو یکجا کریں
متغیر کے لئے حل کریں
عنصر
توسیع
اجزاء کا جائزہ لیں
لکیری مساوات
دوطرفہ مساوات
عدم مساوات
مساوات کے نظام
میٹرکس
Trigonometry
آسان بنائیں
اندازہ
Graphs
مساوات کو حل کریں
حسابان
ڈیریویٹوز
لازمی طور پر
حدود
الجبرا ان پٹ
Trigonometry Inputs
کیلکولس ان پٹ
Matrix ان پٹ
بنیادی
الجبرا
trigonometry
حسابان
شماریات
میٹرکس
کریکٹر
جائزہ ليں
0
کوئز
Limits
5 مسائل اس طرح ہیں:
\lim_{ x \rightarrow 0 } 5x
ویب سرچ سے اسی طرح کے مسائل
Prove that for any c \neq 0 \lim_{x \rightarrow c}{h(x)} does not exist and that \lim_{x \rightarrow 0}{h(x)} does exist.
https://math.stackexchange.com/questions/334631/prove-that-for-any-c-neq-0-lim-x-rightarrow-chx-does-not-exist-and
Hint: take one sequence that contains only rationals and another one that contains only irrationals (both tending to c\ne 0). For the case of c=0, you can use e.g. that h is continuous at 0 ...
Proofs regarding Continuous functions 1
https://math.stackexchange.com/questions/526691/proofs-regarding-continuous-functions-1
The proof of part a) needs to be modified a bit. You have used the logic that if N \leq f(x) \leq M then xN \leq xf(x) \leq xM. This holds only when x \geq 0. It is better to change the argument ...
Use L'Hopital's with this problem?
https://math.stackexchange.com/questions/1419122/use-lhopitals-with-this-problem
Let \displaystyle y=\lim_{x\rightarrow 0^{+}}\left(\frac{1}{x}\right)^{\sin x}\;, Now Let x=0+h\;, Then \displaystyle y=\lim_{h\rightarrow 0}\left(\frac{1}{h}\right)^{\sin h} So \displaystyle \ln(y) = \lim_{h\rightarrow 0}\sin (h)\cdot \ln\left(\frac{1}{h}\right) = -\lim_{h\rightarrow 0}\sin h\cdot \ln(h) = -\lim_{h\rightarrow 0}\frac{\ln(h)}{\csc (h)}\left(\frac{\infty}{\infty}\right) ...
Calculate: \lim_{x \to 0 } = x \cdot \sin(\frac{1}{x})
https://math.stackexchange.com/questions/1066434/calculate-lim-x-to-0-x-cdot-sin-frac1x
Your proof is incorrect, cause you used incorrect transform, but it has already been stated. I'll describe way to solve it. \lim_{x \to 0}\frac{\sin(\frac{1}{x})}{\frac{1}{x}} \neq 1 Hint : ...
Prove that f(x) is bounded. Please check my proof.
https://math.stackexchange.com/q/1052420
Here is another approach: Let L_0 = \lim_{x \downarrow 0} f(x), L_\infty = \lim_{x \to \infty} f(x). By definition of the limit we have some \delta>0 and N>0 such that if x \in (0, \delta), ...
Complex Function limit by investigating sequences
https://math.stackexchange.com/questions/1915934/complex-function-limit-by-investigating-sequences
If a limit as z \to 0 exists, one should be able to plug in any sequence \{ z_n \} going to zero and get the same limit. Limits of sequences are generally easier to work with. So in this case if ...
مزید اشیا
حصہ
نقل
کلپ بورڈ پر کاپی کیا گیا
اسی طرح کے مسائل
\lim_{ x \rightarrow 0 } 5
\lim_{ x \rightarrow 0 } 5x
\lim_{ x \rightarrow 0 } \frac{2}{x}
\lim_{ x \rightarrow 0 } \frac{1}{x^2}
اوپر کی طرف واپس جائیں